How can the arc length of a cycloid be calculated using parametric equations?

Click For Summary
The discussion focuses on calculating the arc length of a cycloid using parametric equations. It begins with an integration involving secant squared and explores the implications of analyticity in complex functions. The parametric equations for the cycloid are provided, along with the derivatives needed for arc length calculation. A question arises about solving the integral for arc length, specifically whether it can be simplified to s^2 = 2a^2 ∫(1 - cos(θ)) dθ. The thread seeks clarification on the correctness of the substitution and integration steps involved in deriving the arc length formula.
John O' Meara
Messages
325
Reaction score
0
Integrate \int_C \sec^2 z dz \ \mbox{any path from } \ \frac{\pi}{4} \mbox{ to } \frac{\pi\iota}{4} \\ \sec^2 z = \frac{1}{\cos^2 z} \ \mbox{ which is equal to } \ \frac{1}{2(1+\cos 2z)} \\ Therefore \frac{2}{1+\cos 2z} = \frac{2}{1 + \cos2x\cosh2y -\iota \sin2x\sinh2y}\\
How do you split this fraction up to differentiate it w.r.t., u_x \mbox{,} v_y \\ i.e., the Cauchy - Riemann equations? It seems to me that sec^2 z is not analytic.
Therefore integrate as follows:z(t) = t + (\frac{\pi}{4} -t)\iota \ \frac{dz}{dt} = 1 - \iota \\ Therefore the integral is \int_0^{\frac{\pi}{4}} \sec^2(t+(\frac{\pi}{4}-t)\iota)(1 - \iota ) dt\\. Am I correct so far? Is z substituted in \sec^2z \ done correctly. Thanks for helping.
 
Physics news on Phys.org
If f(z) is analytic, then f(z)^2 is analytic and 1/f(z) is analytic (away from poles). Since cos(z) is analytic, there is no point in checking CR for sec(z)^2. They will be satisfied.
 
Cycloid question

The parametric equations of a cycloid are x=a(\theta - \sin\theta) \mbox{ and } y=a(1-\cos\theta) \\ where a is a constant. Show that S^2=8ay, where s is the arc length measured from the point theta = 0.

\frac{dx}{d\theta} = a(1 - \cos\theta) \ \frac{dy}{d\theta} = a\sin\theta \\. Then \int \sqrt{a^2(1-\cos\theta)^2 + a^2\sin^2 \theta} d\theta \\. Now this gives \int \sqrt{2}a\sqrt{1 - \cos\theta} d\theta\\. I could do with a hint as to, how to solve this integral. Can I simply say s^2=2a^2 \int (1 - \cos\theta)d\theta \\. I certainly think not. Thanks for helping.
 
Question: A clock's minute hand has length 4 and its hour hand has length 3. What is the distance between the tips at the moment when it is increasing most rapidly?(Putnam Exam Question) Answer: Making assumption that both the hands moves at constant angular velocities, the answer is ## \sqrt{7} .## But don't you think this assumption is somewhat doubtful and wrong?

Similar threads

Replies
13
Views
2K
Replies
6
Views
2K
Replies
1
Views
1K
  • · Replies 9 ·
Replies
9
Views
2K
  • · Replies 3 ·
Replies
3
Views
2K
Replies
23
Views
2K
  • · Replies 3 ·
Replies
3
Views
2K
Replies
32
Views
3K
Replies
9
Views
2K
  • · Replies 13 ·
Replies
13
Views
3K